LSAT and Law School Admissions Forum

Get expert LSAT preparation and law school admissions advice from PowerScore Test Preparation.

 AylixW
  • Posts: 14
  • Joined: Jul 11, 2012
|
#4548
Hi, I have a question about #12 from the Logical Reasoning II section-June 2009 Test.

[Question Removed by Admin -- LSAT questions cannot be posted per LSAC question disclosure rules. Just referencing the PrepTest number, Section, and Question Number as you did in the post title gives us all the info we need! ]

I understand that it is a sufficient necessary question and therefore diagrammed it out. However, I am still having a hard time understanding why E is the correct answer. My guess is that E is correct because: No international police force --> no effective law, therefore: effective law --> international police force. However, I don't fully understand why for example, A is incorrect since "No one" and "Only" are both strong language. If you could please explain why the rest are wrong and E is right that would be great!

Thanks!
User avatar
 Dave Killoran
PowerScore Staff
  • PowerScore Staff
  • Posts: 5853
  • Joined: Mar 25, 2011
|
#4551
Hi Aylix,

In (E), this is what the author said at the end of the argument: "...there is currently no international police force. Hence, what is called "international law" is not effective law." Think about the meaning and relationship of these two statements for a moment. The author is basing that conclusion idea on the lack of an international police force, and assuming that such a force is needed for for enforcement. Why does the author assume this? Because if other methods were out there for enforcing the law, the argument wouldn't make sense (for example, what if domestic police forces in each country could effectively enforce the international laws? ).

To help clarify this idea, let me use the analogy of, "We don't have any power in the house for lighting, so we can't read anything in the house." Here, my argument rests on assuming that the presence of power and electric lights is needed for us to read. But, what if it is daytime? Or if we have candles, or flashlights?

Going back to the original argument and answer choice (E), hopefully the relationship is somewhat clearer. Because this is an Assumption question, you can also negate the answer, and that will reflect a bit of the discussion above.

Answer choice (A) is a really attractive wrong answer. It involves some of the concepts from the stimulus, and seems powerful enough to be a strong contender. Conditionally, it appears as: Obey command :arrow: obedience mechanism.

One issue here that people often note is the idea of a "command" versus the concept of "laws" in the stimulus. The other issue in this answer is one of quantity. Is it really that the author believes that no one will obey, or just that at least some won't? (And remember, assumptions are about minimalist ideas, in the sense that it is the very least that the author must assume, so this quantity difference is important).

Again, this difference can more easily be seen by negating the answer. How would the author respond if you said: "Some people will obey a command even without an obedience enforcement mechanism." He or she would probably say, "That's great that some people will do that, but for a law to be effective in a society, we still need that enforcement mechanism." Basically, it's not about that some people will obey but more about that some people won't. In that sense, this answer would be more attractive if it read: "Some people won't obey a command unless mechanisms exist to compel obedience." If you negate that, it means that everyone will obey a command even without enforcement mechanisms. That would cause the author a lot more problems.

Please let me know if that helps. Thanks!
 Chulam
  • Posts: 1
  • Joined: Jun 06, 2013
|
#9667
Dave Killoran wrote:To help clarify this idea, let me use the analogy of, "We don't have any power in the house for lighting, so we can't read anything in the house." Here, my argument rests on assuming that the presence of power and electric lights is needed for us to read. But, what if it is daytime? Or if we have candles, or flashlights?
Please let me know if that helps. Thanks
!
I think the flashlight will more impressive for reading in the house than candles. Candles can't be shined more than flashlights.
 jessicamorehead
  • Posts: 84
  • Joined: Jul 07, 2017
|
#45651
Okay, I know this is an assumption question and not a justify one, based on the question stem's wording. I chose C because I saw the argument as comparing two different things (regular law vs international law), and C helped put those two things on an equal playing field. If you negate C and there are other differences between international law and the law of an individual society besides the former's lack of an effective enforcement mechanism, then it would weaken the argument because there could be another reason as to why international law is not effective (aka it's NOT the lack of international police force). What am I doing wrong here? Can someone explain why C is wrong and E is right?

Thank you!
 Malila Robinson
PowerScore Staff
  • PowerScore Staff
  • Posts: 296
  • Joined: Feb 01, 2018
|
#45932
Hi Jessica,
Answer C says: : "The only difference" but the stimulus never says this is the only thing that is different, it just names a particular difference; there could be others. So if you negate C and say that 'there are some other differences', it wouldn't hurt the argument.
With Answer E, if we negate it and say that 'something other than an international police force could effectively enforce international law' then the argument in the stimulus that says: 'police enforce society's laws, so international police must enforce international society's laws', would be flawed. The negation of E hurts the stimulus, so it is the correct answer.
Hope that helps!
-Malila
 lsat_novice
  • Posts: 29
  • Joined: May 29, 2018
|
#47388
I thought B was the right answer because negating it would be, "If an international police force were established, then so-called international law would not become effective law." Doesn't that weaken the conclusion?
 James Finch
PowerScore Staff
  • PowerScore Staff
  • Posts: 943
  • Joined: Sep 06, 2017
|
#47604
Hi LSAT Novice,

This question gives us a conditional relationship:

Effective Law (EL) :arrow: Effective Enforcement Mechanism (EEM),

and the contrapositive,

EEM:arrow: EL

The stimulus then tells us that police are one form of EEM. It then concludes that because there is no international police force, international law isn't effective, or:

PoliceInternational :arrow: ELInternational

From here, we can see that in order to justify the conclusion, we need to show that lack of a police force leads to a lack of an effective enforcement mechanism, or:

PoliceInternational :arrow: EEMInternational

Answer choice (E) provides this missing link in the conditional chain, and is the correct answer.

Answer choice (B) gives us a Mistaken Negation of the conclusion, or:

PoliceInternational :arrow: ELInternational

Hope this clears things up!
 lsat_novice
  • Posts: 29
  • Joined: May 29, 2018
|
#47622
Thanks! It makes more sense now.
 Tara
  • Posts: 1
  • Joined: Jul 18, 2018
|
#48077
I still do not follow how B is not an assumption required by the argument even though I understand E to be the correct answer.

Isn't the validity of E as the correct answer implied in answer b. Since only an international police force could effectively enforce international law based on the argument, then wouldn't it be valid to say that if an international police force were established, then international law would become effective law? :-?
User avatar
 Jonathan Evans
PowerScore Staff
  • PowerScore Staff
  • Posts: 726
  • Joined: Jun 09, 2016
|
#48157
Hi, Tara,

Great question! The reason why answer choice (B) is not a necessary assumption has to do with peculiarities of conditional statements as assumptions.

Answer choice (B) gives a Mistaken Reversal™ of the author's assumption. The author assumes that for international law to be effective, there must be an international police force. This may be diagrammed thus:
  • Effective International Law :arrow: International Police Force
Answer choice (B) gives us the converse of this statement:
  • International Police Force :arrow: Effective International Law
The author does not have to believe that an international police force would be sufficient to ensure we have effective international law. The author only must believe that an international police force is necessary for effective international law.

In other words, it could be possible, even with an international police force, that international law be ineffective. The author views an international police force as a precondition for effective international law; however, there could be other factors that would make international law ineffective, even if there were an international police force.

I hope this helps!

Get the most out of your LSAT Prep Plus subscription.

Analyze and track your performance with our Testing and Analytics Package.